2
$\begingroup$

(asked previously in MSE here)

In the course of a calculation, I have met the following complicated identity. Let $A$ and $a$ be positive integers. Then I believe that $$ \sum_{B\ge A,b\ge a} (-1)^{a+b}{b\choose a} {B-1\choose A-1}\frac{(B-1)!b!}{(B+b)(x-b)^{(B+b)}}=\frac{(A-1)!a!}{(A+a)(x-A+1)^{(A+a)}},$$ where $x$ is some variable and $(x)^{(a)}=x(x+1)\cdots(x+a-1)$ is the rising factorial. This is what I would like to prove.

Notice how the left hand side in principle has poles at all integer values of $x$, while the right hand side only has poles at integers smaller than $A-1$.

$\endgroup$

1 Answer 1

4
$\begingroup$

Rewriting the l.h.s. of the conjectured identity and using the properties of beta function, we have:

\begin{split} \text{l.h.s.} &= \sum_{B, b} (-1)^{a+b}{b\choose a} {B-1\choose A-1}\frac{1}{(B+b)\binom{B+b-1}b (x+B-1) \binom{x+B-2}{B+b-1} }\\ &= \sum_{B, b} (-1)^{a+b}{b\choose a} {B-1\choose A-1} \int_0^1 \int_0^1 (1-y)^{x-b-1} y^{B+b-1} (1-z)^bz^{B-1}\,{\rm d}y\,{\rm d}z\\ &= \int_0^1 \int_0^1 (1-y)^{x} y^{-2} (z(1-z))^{-1} \sum_{B, b} (-1)^{a+b}{b\choose a} {B-1\choose A-1} (y(1-z)/(1-y))^{b+1} (yz)^{B}\,{\rm d}y\,{\rm d}z\\ &= -\int_0^1 \int_0^1 (1-y)^{x} y^{-2}(z(1-z))^{-1} \left( \frac{y(1-z)}{1-yz} \right)^{a+1} \left(\frac{yz}{1-yz}\right)^{A}\,{\rm d}y\,{\rm d}z\\ &= -\int_0^1 \int_0^1 (1-y)^{x} (yz)^{A-1} (y-yz)^a (1-yz)^{-(A+a+1)}\,{\rm d}y\,{\rm d}z. \end{split}

Substituting $(u,v)=(\frac{1-y}{1-yz},yz)$, or $(y,z) = (1-u(1-v),\frac{v}{1-u(1-y)})$ we further get \begin{split} ... &= \int_0^1 \int_0^1 u^x (1-u)^a v^{A-1} (1-v)^{x-A} \frac1{1-u(1-v)}\,{\rm d}u\,{\rm d}v\\ &= \sum_{i\geq 0}\int_0^1 \int_0^1 u^{x+i} (1-u)^a v^{A-1} (1-v)^{x-A+i}\,{\rm d}u\,{\rm d}v \\ &= \sum_{i\geq 0} \frac{1}{(a+1)A\binom{x+i+a+1}{a+1}\binom{x+i}{A}} = \sum_{i\geq 0} \frac{1}{(a+1)A\binom{A+a+1}A \binom{x+i+a+1}{A+a+1}},\\ &= \frac{x+a+1}{(a+1)A\binom{A+a+1}A (A+a) \binom{x+a+1}{A+a+1}} = \frac{1}{(A+a)^2\binom{A+a-1}{a}\binom{x+a}{A+a}}, \end{split} which matches the r.h.s.

QED

$\endgroup$
6
  • $\begingroup$ Wow, that's some calculation. Thanks, I will put a bounty on it when it is allowed. $\endgroup$
    – Marcel
    Jan 28, 2022 at 16:32
  • 1
    $\begingroup$ If $b\ge x$, then the corresponding beta integral is infinite. How do you deal with this? $\endgroup$ Jan 28, 2022 at 16:33
  • $\begingroup$ However large $x$ is, $b$ can be larger than $x$. I guess some modification of your proof is needed. $\endgroup$ Jan 28, 2022 at 16:38
  • $\begingroup$ @IosifPinelis: I think we need to replace $(x-b)^{(B+b)}$ with $(-1)^{B+b} (B+b)! \binom{b-x}{B+b}$. I'm pretty sure the idea still works in this setting. I'll try to rewrite it more accurately. $\endgroup$ Jan 28, 2022 at 16:50
  • $\begingroup$ @MaxAlekseyev I put the bounty. Did you find the time to address the $b\ge x$ point? $\endgroup$
    – Marcel
    Feb 9, 2022 at 14:58

Your Answer

By clicking “Post Your Answer”, you agree to our terms of service and acknowledge that you have read and understand our privacy policy and code of conduct.

Not the answer you're looking for? Browse other questions tagged or ask your own question.